f(x)=x^2-ax+1(x>0) (1)若f(x)<e^x恒成立,求a的取值范围 (2)若f(x...

बनाया गया: 24 फ़रवरी 2025

का उपयोग करके उत्तर दिया गया O1 Pro द्वारा Chat01

प्रश्न

f(x)=x^2-ax+1(x>0)
(1)若f(x)<e^x恒成立,求a的取值范围
(2)若f(x)(1-2x)<e^(3x)恒成立,求a的取值范围

विचार

Evaluating polynomial growth

I’m working through the inequality x2ax+1<exx^2 - ax + 1 < e^x for all x>0x > 0. As xx increases, exe^x surpasses x2ax+1x^2 - ax + 1, especially for large xx.

Evaluating positivity

I’m analyzing g(x)=exx2+ax1g(x) = e^x - x^2 + ax - 1 and its derivatives to ensure g(x)>0g(x) > 0 for all x>0x > 0. The second derivative reveals a minimum at x=ln2x = \ln 2.

Determining the threshold

I’m establishing M=22ln2+aM = 2 - 2 \ln 2 + a to ensure g(x)>0g'(x) > 0 for all xx by confirming that g(x)g'(x) is strictly increasing.

Determining positivity

As xx approaches 0, the term x(1+a)x(1 + a) becomes the leading term in g(x)g(x). For g(x)g(x) to be positive near 0, aa must be greater than -1.

Analyzing small values

Considering the quadratic term x2/2-x^2/2, I find that if a=1a=-1, g(x)g(x) is negative for small xx. This reveals that a>1a > -1 is necessary.

Ensuring strict increase

OK, let me see—if a>2ln22a > 2 \ln 2 - 2, then g(x)g(x) stays positive after 0. This keeps g(x)g(x) positive for all x>0x > 0.

Confirming conditions

I’m mapping out that for g(x)g(x) to be positive, we need a>2ln22a > 2 \ln 2 - 2. This ensures g(x)g'(x) is strictly increasing and g(x)g(x) stays positive for all x>0x > 0.

Identifying critical points

OK, let's think it through: x=ln2x = \ln 2 is a critical point where g(x)g'(x) reaches a minimum. This shows the function is concave up for x>ln2x > \ln 2 and concave down for x<ln2x < \ln 2.

Analyzing the derivative

I’m thinking through the implications of g(x)g'(x) having a minimum at x=ln2x = \ln 2. This suggests that for M<0M < 0, g(x)g'(x) might turn negative for some x>ln2x > \ln 2.

Confirming initial observations

I’m verifying that g(0)=0g(0) = 0 and checking g(x)g(x) > 0 for x>0x > 0. Since g(0)=0g(0) = 0, now we need g(0)g'(0) to determine if g(x)g(x) is increasing from this point.

Assessing initial conditions

OK, let's see. For g(x)g(x) to stay positive near 0, aa must be greater than -1. This directs the rising slope of g(x)g(x) as xx increases from 0.

Analyzing critical points

I'm piecing together critical points by solving g(x)=ex2x+a=0g'(x) = e^x - 2x + a = 0. This compromises the transcendental equation but simplifies with h(x)=ex2xh(x) = e^x - 2x, noting convex behavior.

Investigating function bounds

I'm working out that h(x)h(x) spans from about 0.6137 at x=ln2x = \ln 2 to \infty. This suggests the minimum value of h(x)h(x) in the interval [0,+)[0, +\infty) is 0.6137\approx 0.6137.

Arriving at aa constraint

OK, let me see. Requiring h(x)+a=0h(x) + a = 0 yields a0.6137...-a \leq 0.6137..., or a2ln22a \leq 2 \ln 2 - 2, ensuring solutions for x0x \geq 0.

Upholding positivity

For a>2ln22a > 2 \ln 2 - 2, g(x)=ex2x+ag'(x) = e^x - 2x + a remains positive, ensuring g(x)g(x) is strictly increasing and stays positive for x>0x > 0.

Reassessing the implications

I’m thinking through the scenario where a<2ln22a < 2 \ln 2 - 2. If g(x)=0g'(x) = 0 has solutions in (0,)(0, \infty), g(x)g(x) might turn negative, breaking the condition.

Mapping out solutions

OK, let's analyze the solutions for h(x)=ah(x) = -a given h(x)h(x)'s single minimum. Confusion might arise if a-a exceeds 0.6137, leading to variable outcomes.

Identifying constraints

I'm thinking through the solutions for h(x)=ah(x) = -a based on aa's relationship to 0.6137. This helps confirm consistency and understand how many solutions exist for a<2ln22a < 2 \ln 2 - 2.

Assessing sign shifts

Checking how g(x)g'(x) changes from positive to negative at x1x_1 involves analyzing g(0)=1+ag'(0) = 1 + a. For g(x)g(x) to stay positive near 0, aa must be greater than -1.

Evaluating critical points

I’m checking the second derivative g(x)g''(x) to identify local maxima and minima of g(x)g(x). Ensuring the local minimum is above 0 keeps g(x)g(x) positive for all x>0x > 0.

Ensuring positivity

I’m working through the inequality g(x2)=x22+(2+a)x2(a+1)>0g(x2) = -x2^2 + (2 + a) x2 - (a + 1) > 0, with ex2=2x2ae^{x2} = 2x2 - a, to confirm the local minimum at x2x2 is above 0.

Ensuring positive slope

I’m thinking about how g(x)g(x) should stay above 0 for x>0x > 0, considering g(0)=0g(0) = 0 and a positive slope at 0. This leads me to focus on keeping a>1a > -1.

Assessing scenarios

I'm thinking about g(x)<0g'(x) < 0 for 1<a<2ln22-1 < a < 2 \ln 2 - 2 and examining the local minimum to ensure g(x)>0g(x) > 0 for x>0x > 0. This involves solving a tricky transcendental equation.

Identifying potential

I'm gathering info on boundary conditions, aiming for the minimal aa where the local min is exactly 0, ensuring g(x)>0g(x) > 0 for x>0x > 0. This involves solving two intertwined equations for aa and x2x2.

Formulating new ideas

I’m working through the equations, simplifying and grouping terms to address the boundary condition for aa. This approach ensures that g(x)>0g(x) > 0 for x>0x > 0.

Verifying and comparing

I checked if x2=1x2 = 1 was a solution, calculated aa as 2e2 - e, and compared it to 2ln222 \ln 2 - 2. This revealed aa is indeed smaller.

Validating and comparing

I confirmed x2=1x2 = 1 as the local minimum and compared a=2ea = 2 - e to 2ln222 \ln 2 - 2, finding that 2e2 - e is indeed smaller.

Determining the solution

OK, let me see. Below ln2\ln 2, the first solution is smaller. This confirms the boundary condition: a>2ea > 2 - e ensures g(x)>0g(x) > 0 for x>0x > 0.

Limiting the options

It seems that the condition a>2ea > 2 - e is crucial for keeping g(x)g(x) positive for x>0x > 0, as it’s more stringent than a>1a > -1.

Mapping out intervals

I see that if a>0.6137a > -0.6137, there are no real roots and g(x)g'(x) is positive, ensuring g>0g > 0 for x>0x > 0.

Identifying the solution

It’s becoming clear that for g(x)>0g(x) > 0 for x>0x > 0, we need a>2ea > 2 - e. This ensures the local minimum is strictly positive.

Determining a's threshold

Ensuring f(x)<exf(x) < e^x for x>0x > 0 requires a>2ea > 2 - e, guaranteeing g(1)>0g(1) > 0 isn't ee.

Pinpointing the range

I've figured out the final range for aa to ensure both positivity and the boundary condition: a>2ea > 2 - e, around -0.718, is the least restrictive.

Assessing the polynomial-exponential dynamics

I’m analyzing when the polynomial (x2ax+1)(12x)(x^2 - ax +1)(1-2x) consistently stays below the exponential e3xe^{3x} for x>0x > 0 by defining h(x)h(x).

Transforming the polynomial

I’m expanding (x2ax+1)(12x)(x^2 - ax + 1)(1-2x) by distributing and combining like terms, considering the sign of each term for a simplified form.

Focusing on early behavior

I’m analyzing h(x)h(x) near x=0x = 0 by calculating the slope at that point. This helps determine if h(x)h(x) consistently stays positive.

Pulling together

I’m expanding e3xe^{3x} near x=0x = 0 and simplifying term by term. This helps reveal h(x)h(x)'s behavior as xx approaches zero.

Determining positivity

I’m digging into h(x)h(x) by examining the linear term (a+5)x(a+5)x near zero. Ensuring a>5a > -5 guarantees h(x)h(x) stays positive for tiny x>0x > 0.

Analyzing derivatives

I’m distinguishing the first and second derivatives of h(x)h(x). The first derivative, h(x)h'(x), and the second derivative, h(x)h''(x), both inform us about h(x)h(x)'s behavior.

Tracking progress

I’m noting that h(x)h(x) starts positive when a>5a > -5 by examining h(0)h(0) and the slope at x=0x = 0.

Confirming local minimum

I’m confirming that at a=5a = -5, h(x)h(x) has a positive local minimum at x=0x = 0, ensuring h(x)h(x) stays non-negative for small x>0x > 0.

Pinpointing the solution

I’m analyzing that h(x)h(x) needs to stay strictly positive for x>0x > 0. Although the slope at x=0x = 0 is positive, h(x)h(x) might dip below zero later on.

Identifying key values

I’m working through special values like x=12x = \frac{1}{2} and x=1x = 1 to ensure h(x)h(x) remains positive. These values don't impose significant restrictions on aa.

Identifying critical points

I’m pinpointing the need for aa to exceed a threshold to ensure h(x)>0h(x) > 0 for x>0x > 0. This involves setting h(x0)=0h'(x_0) = 0 and solving.

Solving boundary conditions

I'm mapping out a system to find aa for h(x)>0h(x) > 0 by setting h(x0)=0h'(x_0) = 0 at a critical point x0x_0 and aiming for a local minimum with h(x0)>0h''(x_0) > 0.

Evaluating boundary conditions

I'm calculating aa by simplifying the equation at x0=1x_0 = 1. The derived value is 22.0855, suggesting it could be a significant boundary for aa.

Confirming boundary conditions

OK, let me see. By substituting x0=1x_0 = 1 into the equation, I'm aligning with the derived value for aa, verifying the boundary for aa.

Validating consistency

Ensuring consistency, I confirm by simplifying h(1)h(1) in terms of e3e^3 and aa, verifying that both boundary and derivative conditions align perfectly.

Verifying local minimum

I’m calculating h(1)h''(1) for a=e3+2a = e^3 + 2. Since h(1)h''(1) is positive, it confirms that x=1x = 1 is a local minimum for h(x)h(x).

Confirming positive slope

I’m checking that for a=e3+2a = e^3 + 2, h(x)h(x) has a positive slope at x=0x = 0, indicating it remains nonnegative after x=1x = 1.

Evaluating positivity

I'm thinking through the behavior of h(x)h(x) for a=e3+2a = e^3 + 2, focusing on x0x \geq 0. Ensuring h(x)h(x) remains nonnegative after x=1x = 1.

Confirming boundary conditions

I'm making sure the boundary conditions are satisfied by checking if maintaining a<e3+2a < e^3 + 2 ensures the strict inequality is upheld.

Reviewing boundary conditions

I'm noting that if a>e3+2a > e^3 + 2, it ensures h(x)>0h(x) > 0 for all x>0x > 0, which aligns with the strict inequality condition.

Piecing together the puzzle

I’m digging into the sign of a2a - 2 and how it relates to e3e^3. It seems there's a contradiction if a>e3+2a > e^3 + 2.

Reassessing a's condition

OK, let's ensure that a<e3+2a < e^3 + 2 holds. I also check that a5a \geq -5 to prevent h(x)h(x) from turning negative.

Testing the limits

Checking the inequality at a=5a = -5 to ensure h(x)h(x) stays nonnegative for all x>0x > 0. This helps validate if the function meets the condition effectively.

Analyzing function behavior

Taking a closer look at a=5a = -5, I find h(x)>0h(x) > 0 for x>0x > 0. Exploring critical points to prevent h(x)h(x) from becoming nonpositive.

Examining critical points

I’m thinking through various values like x=2x = 2 and x=12x = \frac{1}{2}, ensuring the polynomial stays below e3xe^{3x}. Checking for significant constraints or potential violations.

Evaluating constraints

Considering large xx and the polynomial's sign, I notice the need for a simpler bounding method and the likelihood of a single local minimum in x>0x > 0.

Piecing together constraints

I’m starting to see that aa needs to be at least 5-5 and strictly less than e3+2e^3 + 2. This confirms aa is within [5,e3+2)[-5, e^3 + 2).

Testing boundary cases

Investigating a=5a = -5 to ensure h(x)>0h(x) > 0 for x>0x > 0. Checking critical points and slopes to confirm the inequality holds and h(x)h(x) doesn’t become nonpositive for any x>0x > 0.

Investigating potential

I'm curious if a=5a = -5 might work, given that h(x)h(x) stays nonnegative near zero and for large xx. We'll see if a local minimum at x>0x > 0 might cause issues.

Wondering about constraints

I’m curious if the local minimum for a5a \geq -5 stays positive. Considering if a specific xx might cause h(x)=0h(x) = 0 when a=5a = -5.

Ensuring non-negativity

I’m confirming no negative values for h(x)h(x) when a5a \geq -5 by substituting specific xx values. The growth of e3xe^{3x} ensures it remains non-negative.

Assessing boundary impact

I’m thinking through whether a=5a = -5 is feasible, given h(x)>0h(x) > 0 for x>0x > 0. The derivative and second derivative suggest x=0x = 0 is a local minimum, with h(x)h(x) just above zero for x>0x > 0.

Reasoned for 14m 14s

उत्तर

Solution Outline

We are given a quadratic function

f(x)  =  x2    ax  +  1,x>0f(x) \;=\; x^2 \;-\; a\,x \;+\; 1,\quad x>0

and we wish to determine for which values of the parameter aa the following two inequalities hold for all x>0x>0:

  1.   f(x)<ex\;f(x)<e^x
  2.   f(x)(12x)  <  e3x.\;f(x)\,\bigl(1-2x\bigr)\;<\;e^{3x}.

Below is a step-by-step analysis for each part.


Part (1): f(x)<exf(x)<e^x for all x>0x>0

We want

x2    ax  +  1  <  exex(x2ax+1)g(x)>0for all x>0.x^2 \;-\;a\,x \;+\;1\;<\;e^x \quad\Longleftrightarrow\quad \underbrace{e^x - \bigl(x^2 - a\,x +1\bigr)}_{\displaystyle g(x)}>0 \quad\text{for all }x>0.

Define

g(x)  =  ex    x2  +  ax    1.g(x)\;=\;e^x \;-\;x^2 \;+\;a\,x \;-\;1.

We require g(x)>0g(x)>0 for all x>0x>0. A standard approach is:

  1. Check behavior as x+x\to +\infty
    Since exe^x grows faster than any polynomial, for large xx, exe^x will dominate x2ax+1x^2-a\,x+1. Thus for sufficiently large xx, g(x)>0g(x)>0 automatically. No upper bound on aa arises from the xx\to\infty behavior.

  2. Check behavior as x0+x\to 0^+
    Although the domain is x>0x>0, it helps to see how g(x)g(x) behaves very close to 00. Direct substitution at x=0x=0 gives

    g(0)  =  e00+a01  =  11  =  0.g(0)\;=\;e^0 -0 +a\cdot 0 -1 \;=\;1-1\;=\;0.

    To see whether g(x)g(x) is above 00 immediately to the right of 00, look at
    g(0)=ddxg(x)x=0\,g'(0)=\frac{d}{dx}g(x)\bigl|_{x=0}.

    • We have g(x)  =  ex    2x  +  a,g'(x)\;=\;e^x \;-\;2x \;+\;a, so g(0)  =  e00+a  =  1+a.g'(0)\;=\;e^0 -0 +a\;=\;1+a.
    • For g(x)g(x) to become positive for small x>0x>0, we need g(0)>0g'(0)>0, i.e.\ 1+a>0a>1.1+a>0 \quad\Longrightarrow\quad a>-1.
    • If a1,a\le -1, then immediately for small positive xx, g(x)g(x) would dip below 0 and fail the strict inequality g(x)>0.g(x)>0.

    Hence a necessary condition so far is

    a>1.a>-1.
  3. Check for possible “local minima” of gg on (0,)(0,\infty).
    Define

    g(x)  =  ex2x+a,g(x)  =  ex2.g'(x)\;=\;e^x -2x +a,\quad g''(x)\;=\;e^x -2.
    • The second derivative g(x)=0g''(x)=0 at x=ln2x=\ln 2. For x<ln2x<\ln 2, we have ex<2e^x<2, so g(x)<0g''(x)<0 (the slope g(x)g'(x) is decreasing there). For x>ln2x>\ln 2, g(x)>0g''(x)>0 (the slope g(x)g'(x) is increasing there).

    • Thus g(x)g'(x) achieves its unique minimum value at x=ln2x=\ln 2. That minimum value is

      g(ln2)  =  eln22(ln2)+a  =  22ln2+a.g'(\ln 2)\;=\;e^{\ln 2}-2(\ln 2)+a \;=\;2 -2\ln 2 +a.

      Denote M=22ln2+a.M=2-2\ln 2 +a.

    • If M>0M>0, i.e.\ 22ln2+a>02-2\ln 2 +a>0, then g(x)>0g'(x)>0 for all x>0x>0. Hence g(x)g(x) is strictly increasing on (0,)(0,\infty). Since g(0)=0g(0)=0, a strictly increasing gg means g(x)>0g(x)>0 for all x>0x>0.

      Thus if

      22ln2+a>0a>2ln22,2 -2\ln 2 +a>0 \quad\Longleftrightarrow\quad a>2\ln 2 -2,

      then automatically g(x)>0g(x)>0 for all x>0x>0.

    • What if a<2ln22a<2\ln 2 -2? Then g(x)g'(x) has two positive roots, and gg typically rises, then falls, then rises again. One checks (by more detailed analysis) that the key boundary arises when the local minimum of gg just touches 00. Solving that boundary case leads to

      a  =  2e(approximately 22.71828=0.71828).a\;=\;2 - e\quad(\text{approximately }2 -2.71828\,=\,-0.71828\dots).

      In fact one finds that if

      a    2e,a\;\le\;2-e,

      then g(x)g(x) fails to stay strictly positive for all x>0.x>0.

Putting it all together carefully (and checking numerically around x=1x=1 as a “test point”), one sees:

  • To ensure g(x)>0g(x)>0 for all x>0x>0, the sharp condition that emerges is a>2e.a > 2-e. Indeed, at a=2ea=2-e, you get equality at x=1x=1. For any a2e,a\le 2-e, there is an x>0x>0 where f(x)=ex.f(x)=e^x.

Final answer for Part (1)

The range of a is a>2e.\boxed{\,\text{The range of $a$ is }a>2-e.\,}

Part (2): (x2ax+1)(12x)  <  e3x\bigl(x^2 -a\,x +1\bigr)\bigl(1-2x\bigr)\;<\;e^{3x} for all x>0x>0

Now we want

(x2ax+1)(12x)  <  e3x,(x^2 -a\,x +1)\,(1-2x)\;<\;e^{3x},

for every x>0x>0. Define

H(x)  =  e3x    (x2ax+1)(12x).H(x) \;=\; e^{3x}\;-\;\bigl(x^2 -a\,x +1\bigr)\,\bigl(1-2x\bigr).

We want H(x)>0H(x)>0 for all x>0x>0. First expand the product:

  • (x2ax+1)(12x)  =  x2ax+1    2x(x2ax+1)\,(x^2 -a\,x +1)(1-2x)\;=\;x^2 -a\,x +1 \;-\;2x\,(x^2 -a\,x +1).
  • Carefully expanding: (x2ax+1)(12x)=x2ax+1    2x3+2ax22x, (x^2 -a\,x +1)(1-2x) =x^2 -a\,x +1 \;-\;2x^3 +2a\,x^2 -2x, which simplifies to 2x3  +  (1+2a)x2    (a+2)x  +  1. -2x^3 \;+\;(1+2a)\,x^2 \;-\;(a+2)\,x \;+\;1.

Hence

H(x)=  e3x    [2x3+(1+2a)x2(a+2)x+1]=  e3x  +  2x3    (1+2a)x2  +  (a+2)x    1.\begin{aligned} H(x) &=\;e^{3x} \;-\;\bigl[-2x^3 +(1+2a)x^2 -(a+2)x +1 \bigr]\\ &=\;e^{3x}\;+\;2x^3 \;-\;(1+2a)x^2 \;+\;(a+2)\,x \;-\;1. \end{aligned}

We want H(x)>0H(x)>0 for all x>0x>0. Key steps:

  1. Check behavior as xx\to\infty.
    Since e3xe^{3x} dominates any polynomial, H(x)+H(x)\to+\infty for large xx. Thus no further restriction on aa from the xx\to\infty end.

  2. Check x0+x\to 0^+.
    At x=0x=0,

    H(0)  =  e0    (02a0+1)(120)  =  11  =  0.H(0)\;=\;e^{0}\;-\;\bigl(0^2 -a\cdot0 +1\bigr)\,\bigl(1-2\cdot0\bigr)\;=\;1-1\;=\;0.

    We want H(x)>0H(x)>0 for x>0.x>0. So let us see how HH behaves immediately to the right of 00. Compute

    H(x)  =  ddxH(x)  =  3e3x+6x2    2(1+2a)x  +  (a+2).H'(x)\;=\;\frac{d}{dx}H(x) \;=\;3\,e^{3x} +6x^2 \;-\;2(1+2a)\,x \;+\;(a+2).

    Then

    H(0)  =  3e0+0    0  +  (a+2)  =  3+(a+2)  =  a+5.H'(0)\;=\;3\,e^0 +0 \;-\;0 \;+\;(a+2)\;=\;3 + (a+2)\;=\;a+5.
    • If a+5>0    a>5a+5>0\iff a>-5, then near x=0x=0, the function HH is increasing from the value H(0)=0H(0)=0. Hence for sufficiently small x>0x>0, H(x)H(x) becomes positive.
    • If a+5<0    a<5a+5<0\iff a<-5, then HH would decrease below 0 as soon as x>0x>0, violating the strict inequality.
    • If a=5a=-5, then H(0)=0H'(0)=0. In that borderline case, one can check the second derivative at 0: H(x)  =  9e3x  +  12x    2(1+2a).H''(x)\;=\;9e^{3x} \;+\;12x\;-\;2\,(1+2a). At x=0x=0, H(0)  =  9e0    2(1+2(5))  =  92(110)  =  92(9)  =  9+18  =  27  >  0,H''(0)\;=\;9\,e^0 \;-\;2\,(1+2(-5)) \;=\;9 -2(1 -10) \;=\;9 -2(-9) \;=\;9 +18 \;=\;27 \;>\;0, so x=0x=0 is actually a local minimum of HH with value 00. For x>0x>0, H(x)H(x) then becomes positive. So a=5a=-5 does work.

    From this near-zero analysis, we conclude a necessary (and apparently sufficient) lower bound:

    a    5.a\;\ge\;-5.
  3. Check specific points x>0x>0 to find an upper bound on aa.
    Notice that 12x1-2x changes sign at x=12x=\tfrac12. Also, a simple but crucial test is x=1x=1:

    f(1)  =  12a1+1  =  2a,(121)  =  1.f(1)\;=\;1^2 -a\,\cdot1 +1\;=\;2-a,\quad (1-2\cdot1)\;=\;-1.

    Hence

    f(1)(121)  =  (2a)(1)  =  a2.f(1)\,(1-2\cdot1)\;=\;(2-a)\cdot(-1)\;=\;a-2.

    We want

    (2a)(1)  =  a2  <  e31  =  e3.(2-a)\,(-1)\;=\;a-2\;<\;e^{3\cdot1}\;=\;e^3.

    Since e320.085e^3\approx 20.085, the condition a2<e3a-2 < e^3 means

    a  <  e3  +  2.a\;<\;e^3 \;+\;2.

    Furthermore, if aa reaches exactly e3+2,e^3+2, then at x=1x=1 we get equality:
    (2a)(1)=e3\,(2-a)\cdot(-1) = e^3 .
    That would violate the strict inequality. Hence we must have

    a  <  e3+2.a\;<\;e^3 + 2.

    One can also verify (by looking at derivatives and possible local minima of HH) that x=1x=1 indeed yields the critical upper bound on aa. In fact, if you set a=e3+2,\,a=e^3+2, one finds H(1)=0H(1)=0 and H(1)>0,H''(1)>0, showing that x=1x=1 becomes a local minimum with value 0.0. That “touches” but does not stay below 00. Thus for strict positivity, we need a<e3+2.a<e^3+2.

Final answer for Part (2)

Combining the two bounds:

  • Near 00, we require a5a\ge -5.
  • At x=1x=1, we require a<e3+2.a<e^3+2.
  • Checking consistency shows no further surprises.

Hence the parameter range is

5    a  <  e3+2.\boxed{\, -5 \;\le\;a\;<\;e^3+2\,}.

Final Consolidated Answers

  1. (Part 1)

    f(x)<exfor all x>0a>2e.f(x) < e^x \quad\text{for all }x>0 \quad\Longleftrightarrow\quad \boxed{\,a > 2 - e\,}.
  2. (Part 2)

    f(x)(12x)<e3xfor all x>05    a  <  e3+2.f(x)\,\bigl(1-2x\bigr) < e^{3x} \quad\text{for all }x>0 \quad\Longleftrightarrow\quad \boxed{\,-5 \;\le\; a \;<\; e^3 + 2\,}.

इस प्रश्नोत्तर को साझा करें